Google Answers Logo
View Question
 
Q: Medical Doctor Career, Process of ( Answered 5 out of 5 stars,   7 Comments )
Question  
Subject: Medical Doctor Career, Process of
Category: Reference, Education and News > Education
Asked by: jmswilsn-ga
List Price: $190.00
Posted: 16 Dec 2003 11:29 PST
Expires: 15 Jan 2004 11:29 PST
Question ID: 287766
Hello,

My daughter (Junior in a Private HighSchool) is considering a career
as a medical doctor. Me being of a completely different vegetable, not
knowing much about the process of applying to colleges, meeting
requirements, following guidelines, taking exams, maintaining a record
and an average, and so on -- does not help her at all.

I would like to request a COMPREHENSIVE research into this matter. A
step-by-step  walkthrough of a pre-med student, medschool student, and
post-graduate would be great, if posted in great detail and includes
the bits and pieces of it all, not just a general overview.

Example of a good walkthrough:

"

Second Year of College : PreMed Science Major
- Overview your credits (request your record at the Main/Student
office) and make sure you are meeting the GR (General Requirements) of
your school (can be obtained in the <so and so place>)
[Hint Note: It is very important to ... <useful information>]
- If not yet, consider declaring a major. It would greatly help in
your acceptance into medical school if you take a Science major, such
as Biology, Chemistry, BioChemistry, etc:
-- Look over your grade for the Introductory classes of Biology,
Chemistry and Physics
-- Think over which subject you believe you are most comfortable with
and can pull a good grade with.
-- Visit the college's Biology/Chemistry/etc department and ask the
following questions:
----- "What sort of Biology/Chemistry classes should be taken?"
----- "What are the requirements for <so and so>"
----- "What is the statistical percentage of <so and so>"
----- (etc)
-- Visit your counselor and discuss the matter with him/her.
-- Prepare to give yourself an explanation why you are choosing this or that major.
-- Declare it whenever you are ready
- Look at the requirement classes for your major, visit <so and so> to
gain information about <so and so>
[Hint Note: Biology majors usually .... <this and that>]

"

I'm hoping that I've not asked for too much. Example of a POOR
walkthrough is as follows:

"

4) Pass Highschool with a B+ or above
5) Look for a college that concentrates on medical/science majors
6) Apply to the school and try to get financial aid
7) Signup for classes, you should take basics of Math, Literature...

"

Im sure you get the drift. It would be rather good if someone who
already went through the whole process wrote the answer for me, than
to have someone spend days for research.

The child is planning to attent a college in New York City (she is a
wild bird and I can't seem to control her anymore), she wishes to
become an eye doctor, but is not sure yet.

If you have any questions, please ask them. I would rather have a well
researched answer posted, than a one done in a rush. Good rating and a
generous bonus to those who took their time or have experience, low
rating and harsh words for those who are quick to grab the payment. If
you have gathered some information, or have yourself went through the
whole process and can offer the insight, but the question was already
answered, post a comment and request another question of this kind to
be opened up. You can then post your own answer and get your separate
payment.

Thank you and I'm counting on you!

Request for Question Clarification by boquinha-ga on 16 Dec 2003 20:02 PST
Hello jmswilsn-ga!

What a great question! I applaud what you're doing for your daughter,
especially at this stage in her education. You have a lot of foresight
and that's a good thing, especially when one is considering a career
in medicine. I used to work in the admissions office of a medical
school, so I certainly have some pointers on what kind of things help
an applicant stand out from the crowd. It is a competetive process,
but it can be done!

I see from your very detailed question that you want a lot of
information for ALL stages (that's good--I think it's wise to look
beyond getting into medical school as there is much more to it than
that hurdle alone). You have mentioned that you want information on
residency (post-graduate). There are many different specialties and
depending on the specialty, the number of years required for residency
varies greatly. I'd be happy to shape it however you'd like, giving
you a year by year breakdown or a general overview. You mentioned that
your daughter is interested in being an eye doctor. Would you like me
to gear my answer toward ophthalmology or medicine in general? Some
people go through medical school knowing exactly what they want to
specialize in and others change their minds all through medical school
and even beyond. Peoples' experiences vary greatly depending on their
approach. My husband graduated medical school this past June (we are
in residency) and has always been committed to being a family doctor,
so it was fairly cut and dry during medical school to know what kind
of electives to take in order to gain more knowledge in specific areas
that interest him rather than taking different electives to try to
figure out what to speicialize in (as MANY of his classmates did). If
your daughter is absolutely set on ophthalmology, then I will gear my
answer accordingly. If she is interested in medicine in general and
possibly that specifically, then I will present my research in a more
general manner with some caveats about the specialty. Please let me
know what would be most helpful to you.

In the meantime, I've answered a very similar question that can at
least address the basics for you. Clearly, your question will have
much more detail than this one, but for now, you can get a general
idea from here: http://answers.google.com/answers/threadview?id=258090

If you think of anything else, please let me know. I'll be checking in
here often as I do my research for you.

Sincerely,
Boquinha-ga

P.S. Oh, and since you're into thinking ahead, you may enjoy this
question as well: http://answers.google.com/answers/threadview?id=252363

Clarification of Question by jmswilsn-ga on 16 Dec 2003 20:49 PST
Hell Boquinha,

Thank you for taking interest. Let me try to grab 
everything one by one, if I missed something, please don't 
hesistate and continue researching/answering, I trust 
your judgement!

1) Yes, I prefer all stages. From The first years of college (we tackled
the getting into one part ourselves), to the medical school, to the 
post-grad obtaining residency. You dont have to shape it according 
to any specialty, just an overview of things like "After obtaining your diploma, 
apply .. or.. visit.. or.. make sure to" Etecetera. General information.

2) My daughter is interesting into getting a profession as a 
doctor, she mentioned about how exciting the ophthalmology seems 
to be, but its just my guess that she might become a one. I included 
that information just as an extra, please format your answer as 
general as possible, no need to concentrate on one profession, general 
medicine would suit it better. Thank you.

3) Thank you for the wonderful links to the other 
answers, I see you're quite the pro at all of this.


My deepst respect, and best of luck going through the residency,

Sincerely,

jmswilsn

Clarification of Question by jmswilsn-ga on 16 Dec 2003 20:51 PST
Im sorry, how rude of me, I re-read the Clarification and "Hello" came
out as "Hell", please accept my apologies.

Sincerely,

jmswilsn

Request for Question Clarification by boquinha-ga on 18 Dec 2003 06:51 PST
Hello again, jmswilsn-ga!

Just checking in to let you know that I'm compiling both online
resources as well as personal knowledge to provide you with a thorough
answer. I appreciate your earlier clarifications--they help a great
deal. And please, if you think of anything else, feel free to post
more clarifications to me. I check here frequently as I conduct my
research and organize it all. Also, if your daughter has anything
specific that she'd like to know (or things that she's worried about
or wondering about or . . . ), please let me know and I can also
address those things specifically. Thanks so much! (Oh, and no worries
about the earlier "Hello" thing! I knew what you meant!)

Sincerely,
Boquinha-ga

Request for Question Clarification by boquinha-ga on 20 Dec 2003 09:56 PST
Hello again! I am posting to let you know that I will have your answer
ready for you today. I've spent a lot of time on it and I do believe
that this is, bar none, the longest answer that I have ever written as
a Google Answers Researcher! I am excited to share it with you and
help you and your daughter to have that resource!

Sincerely,
Boquinha-ga
Answer  
Subject: Re: Medical Doctor Career, Process of
Answered By: boquinha-ga on 20 Dec 2003 13:35 PST
Rated:5 out of 5 stars
 
Hello jmswilsn-ga!

Thank you again for an interesting (and thorough) question! Compiling
this answer for you has been a trip down memory lane for me in many
ways (which include both happy and stressful memories). My perspective
on this topic comes from 2 main qualifications. First and foremost, my
husband is a physician and is currently in residency. We got married
during college (undergraduate) and have been going through this whole
process together as a family?we have a lot of experience and are
making more and more all the time! Secondly, I worked in the admission
office of an osteopathic medical university and have some pointers on
what to do and what NOT to do. As we go through these, you will be
picking up some ?lingo? and ?jargon? associated with the whole process
and that should prove helpful in your further research as well as in
your endeavors. We have a lot of ground to cover (over 31 pages in
Word!), so here we go!

To give you an idea of how I?m approaching this topic, I am using your
guidelines as you?ve described in your initial question and following
the subsequent outline:

Allopathic versus Osteopathic Medicine
General Principles
High School/Prior to the Freshman Year

First Year College?Freshman
Second Year College?Sophomore
Third Year College?Junior
Fourth Year College?Senior

Applying to medical schools
Interviewing at medical schools
The wait!
The letters

First Year Med School?MSI
Second Year Med School?MSII
Board Exams
Third Year Med School?MSIII
Fourth Year Med School?MSIV
Board Exams

Applying to residency programs
Interviewing at residency programs
The wait!
Match Day

First Year Residency (Intern)
Board Exams
Second Year Residency (Senior Resident)
Third Year Residency (Senior Resident)

Post-Residency
Specialty Boards

Other Resources

= = = = = = = = = = = = = = = = = = = =

Allopathic versus Osteopathic Medicine

When considering medical school there are two main branches that are
prevalent: Allopathic (MD) and Osteopathic (DO). They both offer the
same medical treatments such as medicines and surgery, but there are
philosophical differences focusing on the ?mind, body, spirit? model
of complete, holistic health. DOs also utilize Osteopathic
Manipulative Therapy (OMT) as an adjunct to ?traditional? medical
practices. Most of the information in this answer is taken from
primarily Allopathic sources, but the general undergraduate course is
very similar. I will include a few links here with information
specific to the Osteopathic Medical field for completion?s sake.

Student Doctor.net?The Network for Every Student Doctor (This is a
fantastic resource, by the way, and I highly recommend it)
http://www.studentdoctor.net/features/do_mdwhich.html
This article addresses the ?DO (Doctor of Osteopathy) vs. MD (Medical
Doctor)? decision.

The American Osteopathic Association
www.aoa-net.org

AACOM (American Association of Colleges of Osteopathic Medicine)
http://www.aacom.org/

= = = = = = = = = = = = = = = = = = = =

General Principles

One of the most important things you can do to improve your medical
school application is to stand out. You do not want to become just
?one of the crowd.? The latest estimates are that less than half of
all applicants to medical schools are accepted, so it is a competitive
proposition. Find ways to distinguish yourself from the other
applicants. This can be achieved in many ways, including selecting a
unique major, gaining interesting life experiences, and holding
various part-time jobs. You are only limited by your imagination. I
will discuss many of these things as I go through this answer.

Also, be sure that this is what you want before devoting a lot of time
and energy (and money!) to this quest. Assuming a ?traditional? time
frame for schooling, it is a MINIMUM of 11 years from day one of
college to the final day of residency. That is a LONG time to be
chasing a dream that isn?t yours. It is a good idea to gain some
experiences in the medical field early on to get an idea of what your
?real? dream may be. If you can shadow a physician or volunteer at a
nursing home (or similar medical facility) while still in high school
that would be great. I?ll discuss more ideas regarding experience
later in the answer.

If you are ABSOLUTELY CERTAIN that you want to attend medical school,
you may consider attending a 6-year program offered by some
universities. Basically, these programs integrate the undergraduate
and graduate education to shave 2 years off of the total experience.
If you are not sure about your career goals, this type of program
allows no flexibility and can hurt you in the end if you change your
mind. The AAMC currently lists 29 such programs available currently
(http://services.aamc.org/currdir/section3/degree2.cfm?data=yes&program=bsmd).

Keep your resume current. Try to update it on a regular basis so that
when application time rolls around you will have very little
?remembering? to do. This timesaving tip is one that has proved to be
helpful time and time again for my husband (and myself), as we have
applied for jobs, graduate schools, and residency programs. It can be
difficult to remember, but worth the time invested.

= = = = = = = = = = = = = = = = = = = =

High School/Prior to Freshman Year

It is important to understand that there are very few required classes
for entrance into medical school, but you must align the courses well
so as to maximize your time in college. The courses are usually
sequential and they may not be offered at a convenient time every
semester, so advanced planning is key. The absolutely required courses
for admission into medical school are:

-One year of Biology
-One year of Physics, with lab
-One year of English
-Two years of Chemistry (basically Inorganic and Organic Chemistry), each with lab

(Association of American Medical Colleges, AAMC,
http://www.aamc.org/students/applying/about/start.htm)
(See also, http://www.bestpremed.com/preMDreq.php)

Every university offers a different variety of courses, but each one
will definitely offer courses that fit into each of the above
categories. Before registering for classes it would be a good idea to
contact an advisor familiar with medical school applications. Some
colleges have a department completely dedicated to Pre-professional
counseling, and some may not. If you are unsure of whether or not
there is such an office, contact someone in the main office for the
College of Biology and ask them who would be the best person to
contact. My husband?s pre-medical advisor had an office within the
College of Agriculture and Biology at our university.

Some medical schools require a semester-long course in Behavioral
Science (Psychology, etc.) and a year of Math, including courses in
Calculus and Statistics. If you are unsure of which medical schools
you are planning to target, it could be wise to take these courses as
part of your General Education (GE) requirements. Each should fulfill
some requirement at most colleges.

A word about GE: I would advise obtaining as many college credits as
possible prior to attending college. These can be obtained through
Advanced Placement classes and exams or college courses themselves. My
husband was fortunate enough to earn 24 credits (or one and a half
semesters at our university) through advanced placement exams in
Calculus and U.S. History. This allowed him to ?skip? history and math
courses at college, freeing up his schedule for other courses.

= = = = = = = = = = = = = = = = = = = =

Freshman Year

* * * * *  Majors: 

There is no great rush to declare an official major, even if you are
absolutely sure what it will be. There are enough GE courses to take
that you will not really have time to dive into major coursework
during the Freshman year anyway, so take this time to explore
interests and get used to the college life. There are no ?required
majors? for entrance into medical school so literally the entire
spectrum is available to you. My husband started as a Political
Science/Philosophy Major, and ended up graduating as a Japanese Major.
Along the way he accumulated almost enough credits to obtain minors in
Philosophy and Chemistry while still finishing his pre-medical
coursework. If you ask him about his choices he has never once doubted
that this was the best possible path for him.

On his site discussing the Premedical Curriculum, Eric Wang, MD, 2002
Graduate of the UCLA Medical School says this about choosing a major:

?Choose a major which best fits your interests. If you try to ?fit?
your interests into biochemistry while in fact they lie with art
history, you will just do poorly in your biochemistry classes, never
explore your passion for art history, and hate college. Medical
schools don't care what major you are! They really don't! But, they do
care that you enjoy what you do, and most importantly, that you do it
well. The application and interviewer can really tell if you are just
doing something for medical school, or if you are doing it because you
love it.?

(http://www.bol.ucla.edu/~ericwang/Curriculum/curric.html)

I tend to agree with this approach and say study what you enjoy and it
will always fit into your future. The important thing is to enjoy your
studies and to do well.

* * * * * Courses:

This is definitely the time to be taking GE courses. Your specific
needs will vary depending upon your college and also what Advanced
Placement credits you will be bringing with you (as I discussed
above). This is a perfect opportunity to acclimate to a college
environment and so I suggest starting with easier courses in which you
are more likely to perform well. From the AAMC site, there are some
interesting statistics about the 2003 applicants and matriculants
(students who are accepted and enter) to medical school:

Science GPA?Applicants: 3.36
Non-Science GPA?Applicants: 3.60
Total GPA?Applicants:  3.47

Science GPA?Matriculants:  3.55
Non-Science GPA?Matriculants:  3.70
Total GPA?Matriculants: 3.62

(http://www.aamc.org/data/facts/2003/2003mcatgpa.htm)

As you can see, those who were accepted and subsequently entered
medical school have GPAs that are about .1 to .2 higher in each
category. Thus it is extremely important to ?get off on the right
foot.? Dr. Wang had this to say about the importance of GPA in
admission to medical school:

?Medical schools would rather see great grades in easy classes than
good grades in harder ones. That is the sad reality of medical school
admissions. Grades are used to determine GPA, and this is one of the
only concrete rankings, so medical schools use them. They have no idea
what the difference between Physics 160 and Physics 1 is, or what
makes Bio 225 so different from Bio 11 at your school. But, they do
know that an A is better than a B. Medical schools are not going to
try and weigh if Bob's B in graduate level physics indicates a better
student than Bill's A in physics for lobotomized mice. They will see:
1.	Bob's physics: B 
2.	Bill's physics: A 
And, that's what they care about most: not the class, but the letter
grade. In fact, on the AMCAS application, your science GPA,
non-science GPA, and overall GPA are given on the front; only upon
closer review in the back can one actually see your classes. So,
that's the sad reality of medical school admissions. I don't agree
with it, and personally, I think learning is more important than
getting slightly better grades, but it's important to understand what
is happening these days.?

(http://www.bol.ucla.edu/~ericwang/Curriculum/curric.html)

I do not advocate taking easier courses to ?pad? your GPA. A
well-rounded education cannot be measured in terms of GPA and test
scores, but I do advocate pacing yourself and setting realistic goals.
You should consider talking with some sort of academic advisor as you
set up your schedule, but here are some suggestions for courses for
the Freshman Year:

-A good first-year writing course. This will be an invaluable tool to
you throughout your education, will usually be a requirement for
graduation, and should fulfill half of your English requirement for
medical schools.

-A physical education course in an area you enjoy. Most schools have a
GE requirement involving PE so why not start now? You can choose from
a variety of courses (even dance and bowling at some institutions) and
can establish a means by which you can ?blow off steam? as you work at
your studies.

-Consider beginning your Physics or Inorganic Chemistry courses. You
probably will want to avoid a lab at this time because they tend to be
very time intensive considering the number of credits you receive (my
husband spent 3-6 hours per week in a lab course for only one credit).
If you are very confident in your abilities you may consider starting
both now, but my husband waited until his Sophomore year to start
Chemistry and was very glad he did so.

-Take the time to take some courses that just seem interesting right
now. This is the perfect time to explore the religions of Eastern
Asia, the great philosophers, or an introduction to the Arts and
Humanities (my major?yes, this is a shameless plug!). The point is,
the opportunity to learn in this type of environment from (hopefully)
world experts in their fields is a (pardon the cliché)
?once-in-a-lifetime opportunity.? You can learn more about possible
majors and also enrich your own life. Remember that every experience
you gain is one more you can apply to your future career.

* * * * * Extracurricular Activities:

This is one area where you can begin to ?stand out? among fellow
medical school applicants. Every applicant will have similar grades
and test scores, but hopefully your volunteer and other
extracurricular activities will be very unique. During the first year
it is a good idea to limit your involvement to one or two activities
for which you have a particular interest. Remember that unfortunate as
it may be, GPAs are very important when applying to medical school.
Pick activities that will provide a needed ?break? from your studies
and that will enrich you. It is never too early to become involved in
volunteer activities, which are becoming necessary in order to become
accepted at a medical school. Most universities will have clubs and
organizations that have community service as a particular focus, so
you may consider joining one of them. Of course you can also create
your own experiences as well. Volunteering at a nursing home or with
an organization servicing the disabled is a good way to not only
provide community service, but also to gain valuable experience in the
medical field.

Dr. Wang has some interesting insight regarding volunteer service:

? I highly recommend against volunteering in a hospital because they
have a glut of volunteers like yourself, and you will most likely have
very little clinical contact. But, you definitely have to get some
kind of exposure to clinical medicine, either by shadowing a doctor,
working in a hospice, taking care of elderly or handicapped persons,
or other such contact. You must prove to the admissions committee that
you know what it is like to work with the sick, aged, and infirm. If
you don't get some exposure, then you'll be stuck when an interviewer
says, ?how do you know you want to be a doctor when you've never been
in a position to see what one does???

(http://www.bol.ucla.edu/~ericwang/Curriculum/curric.html)

In addition to discouraging volunteer work in hospitals, Dr. Wang
discourages working in a lab (unless you really want to). It?s hard
work and can be very tedious if it is not one of your specific
interests. My husband worked in a lab for a short time, but had a
miserable experience since this is not a particular interest of his.

I would encourage getting involved in your university?s Pre-Med Club
(if they have one) so that you can meet other students interested in a
career in medicine. You can watch as upper classmen go through the
testing and application processes, gain insight as they share
interview experiences, and be in touch with resources available to
prospective medical students. They usually arrange service projects,
are aware of volunteer experiences, and can direct you toward
physicians in the community that are excellent resources.

* * * * * Employment:

While employment will diversify your experiences, and provide you with
needed financial resources (medical school is expensive?we?ve got
quite the debt to prove it!), you may want to consider waiting until
your second year of school to start a part-time job. Getting used to
the rigors of college life is difficult, and being able to concentrate
on your studies, extracurricular activities, and social life (yes,
that is important, too!) is a priority during the first year.

= = = = = = = = = = = = = = = = = = = = =

Sophomore Year

* * * * * Majors:

If you haven?t done so already, now would be a good time to declare
and focus on a major. Think about your strengths and weaknesses, along
with your interests and discuss these with an academic advisor,
preferably one within the Pre-medical/Pre-professional department. Be
prepared to discuss your reasoning for selecting a specific major,
along with pros and cons with regards to medical school. Remember that
there is no one major that is ?right? for medical school, but it
should be one that fits with your personal interests and goals.
Looking at data on the 2003 applicants and matriculants
(http://www.aamc.org/data/facts/2003/mcatgpabymaj1.htm), the GPAs and
MCAT scores of matriculants are fairly consistent throughout the
majors listed, so as long as you do well in all of your courses,
science and non-science, you will be a competitive applicant. Your
pre-professional department should have data available for specific
majors at your university. Perusing this information may help you in
your ultimate decision regarding a major.

* * * * * Courses:

Depending on what courses you took during your freshman year and how
many credits you brought to college, your options may be very
different at this time. Remember that the optimal time for taking the
Medical College Admission Test (MCAT) is in April of the year before
you will attend medical school (i.e., most students will take it in
April of their Junior years). Here are a few recommendations (and as
always I recommend consulting with an academic advisor while
scheduling classes):

-Finish your Physics and Inorganic Chemistry courses, along with their
corresponding lab section. This is especially important for Chemistry
since you still have one year of Organic Chemistry with lab to
complete prior to the MCAT. By finishing your Inorganic Chemistry
course now you have your junior year to complete Organic Chemistry.

-Begin your Biology courses. You need a full year of courses and this
is a good time to at least start. Your school may have a Life Sciences
or Biology GE requirement so there is a good overlap on this one. Be
sure to confirm which courses at your institution qualify as ?Biology
Courses? when applying to medical school. Your pre-medical counselor
should have this information. One recommendation is a good Physiology
course. There is a lot of Physiology on the MCAT and a good course can
serve as excellent test preparation.

-Complete as many GE courses as possible. These tend to be fairly time
consuming and relatively hard compared to courses in your major. Get
these out of the way now to free up time later for MCAT study and
medical school applications (these take a lot of time to complete).

-Begin your major. You may not need to take many courses right now if
your major does not have that many hours to complete. My husband
majored in Japanese and because of his prior language experience he
only had 30 or so hours to complete for his major. This is the
equivalent of just under two full semesters at our institution. Majors
in Mathematics and Sciences tend to require more hours so you will
need to get a good start now to graduate in four years. Of course
consult with an advisor in your major?s department to determine the
best timing for your coursework.

-If you still have time, try to take at least one course for
?enrichment.? This can be a great way to obtain necessary credits for
graduation and to also continue exploring fields of interest for you.

-Be on the lookout for any type of Premedical Seminar. Our university
offered premed courses that brought in community physicians as guest
lecturers, discussed specifics regarding application procedures, and
provided other interesting experiences. They also offered one-time
evening seminars that were intended to prepare students for the MCAT
and application process.

* * * * * Extracurricular Activities:

Stay involved if you have been, and get involved if you haven?t.
Admission committees at medical schools will notice if you?ve ?crammed
in? a lot of activities toward application time. They want to see
students that can balance studies, work, extracurricular activities,
etc. over a long period of time, not just a span of a few months. 
It?s not bad to be involved in a variety of activities over your time
in school, nor is it bad to be involved in the same one or two for the
entire four years. Just stay involved in something you enjoy.

An additional activity to become involved in is mentor shadowing
programs. Some schools have formal programs in place and some do not.
If yours does not have one in place, arrange these experiences
yourself. Basically what it entails is following a physician
throughout a full or partial workday in order to gain an appreciation
for what it is that physicians do. Some medical schools insist that
you have a letter of recommendation from a physician so this is an
excellent way to establish a relationship with a physician. I would
recommend shadowing a physician more than once or twice in order to
establish a more significant connection and also to gain more exposure
to the medical field.

According to AAMC extracurricular activities are essential,
?especially those reflecting public or health-related service,
volunteer work, and other evidence of your initiative.?

(http://www.aamc.org/students/considering/gettingin.htm#Look)

* * * * * Employment:

As you approach MCAT time and the application season, it is important
to shore up resources to pay for all of these things. In 2004 the MCAT
will cost $190 and applications begin at $150 for the first school,
and $30 for each additional school to which you will send an
application. There are also fees for filing secondary applications
(which I will discuss later) and the costs associated with travel to
conduct interviews. It all adds up. Any part-time work that you can do
during school and full-time work between semesters will be very
useful. It is also an excellent way to demonstrate that you are
capable of balancing many activities while performing well
academically.

* * * * * MCAT Preparation:

While it is still early to be too worried about the MCAT, it is never
too early to start at least thinking about it. Once you have completed
the required science courses (Physics, Chemistry, and Biology) you
should have enough knowledge to perform well. Even before you have
completed all of these courses you can consult a number of resources
available to students preparing for the MCAT, including complete
practice tests online (see the Official MCAT site
http://www.aamc.org/students/mcat/start.htm). I will discuss more
about the preparation later in this answer.

Of course, Dr. Wang has something to say about MCAT preparations:

?As for when to take the test, I would say, ?As early as possible.?
Most take it in April, but that doesn't mean you have to. Take it as
soon as you've finished the material that is covered on the test (e.g.
organic chemistry, general chemistry, physics, and
physiology/biology). You just want to get it out of the way; it's a
highly stressful time, and you don't want to compound it with other
worries such as thesis research, etc?

(http://www.bol.ucla.edu/~ericwang/Curriculum/curric.html)

= = = = = = = = = = = = = = = = = = = =

Junior Year

* * * * * Major:

By now you should have chosen a major and should also be taking
classes within it. With graduation approaching in two years it?s time
to begin a more concentrated study of your specific major. Discuss
specific plans with your academic advisor within your major so you can
best accomplish your goals. Some colleges even assign a specific
advisor who will follow you throughout the four years so you can be
sure that he or she is familiar with your specific situation.

* * * * * Courses:

By the junior year you should be very familiar with your Premedical
and Major-specific academic advisors. Consult frequently with them
concerning your graduation plan and course selections. I have a few
suggestions at this time:

-Be sure that you?ve completed your science courses. Most students
will have completed Physics and Inorganic Chemistry by the end of the
sophomore year, and have dedicated the junior year to the study of
Organic Chemistry. It is a difficult subject for a lot of students and
is worth the time investment since a lot of questions on the MCAT will
come from Organic Chemistry.

-Ensure that you have completed your Biology requirements, including a
Physiology course. These are ?high yield? in terms of the amount of
material that shows up on the MCAT.

-Complete your English requirement with a semester of Advanced
Writing. Most universities will require this for graduation and more
writing practice is useful as there is a section of the MCAT devoted
to it.

-If you have time, consider another ?enrichment? course. It may or may
not be medical in nature. My husband enjoyed taking philosophy courses
that focused on Bioethics. It satisfied his interest in philosophical
arguments while still broadening his medical knowledge. You will find
that with all of the focus on medical school applications, a little
?enrichment? will go a long way.

* * * * * Extracurricular Activities:

Keep doing them. They don?t get any less important. At this time you
may want to become more involved in a leadership capacity in one of
your activities. This is a good way to stand out on an application,
especially if you have been President or Vice President of a club.
Again, stay within your interests and find ways to stay involved in
medical activities.

* * * * * Employment:

Don?t give up now, but be sure to balance time at work with time for
studies. If you can do it try and get a job in a medical field (Nurse
Assistant, phlebotomist, EMT, etc.). The more practical training you
can get now the better off you will be later. Many applicants to
medical school have medical training of some sort, so any that you can
gain keeps you competitive.

* * * * * MCAT:

First of all, here is the official site:
Official Medical College Admission Test Website
http://www.aamc.org/students/mcat/start.htm


This site gives you dates, updates, and sample questions, among other
helpful things! There are even 4 full-length practice MCATs available
online! Be sure to stay up to date on deadlines for registration for
the test. It is recommended that you take the MCAT in April the year
prior to your anticipated admission in to medical school, so for most
students that translates in to the end of their Junior years. You can
take the test in August, but by the time the results are released, the
application process is well underway. You will find that the advantage
goes to those that get their applications in early, so unless you plan
to apply to medical school the following year, the August test will
only hurt you. The exception is that August is a good time to retake
the MCAT if you feel you performed poorly. Your application will be
complete with your April results, but the medical schools will know
that you are retaking it. As always, discuss your specific plans with
your premedical advisor.

Your preparation should be ongoing throughout the year, and your
current courses will probably be excellent resources. Be sure to allow
yourself plenty of time to brush up on problem areas and take as many
practice exams as possible. There are also official preparation
courses that are available nationwide. It is widely thought that the
courses are not intrinsically more helpful than self-study, but that
depends on the individual. My husband purchased an MCAT review book
and took sample tests and did quite well. Some of his classmates took
review courses with similar results. It is all up to the individual.

And the Frequently Asked Questions about the MCAT can be found here:
http://www.aamc.org/students/mcat/about/faqs.htm

More MCAT Information
http://www.aamc.org/students/mcat/essentials04.pdf

Even more MCAT Information
http://www.bestpremed.com/MCAT.php

The MCAT is an all-day test. I got up and made my husband his favorite
breakfast and gave him a whole bunch of positive, moral support. I met
him for lunch (between the major 2 parts of the exam) and we ate a
picnic lunch on campus (he took it at our undergraduate university).
It was an exhausting exam, but it sure feels good to get it over with!
It?s a big hurdle in the applying-to-medical-school stage!

As a point of interest I will include the 2003 statistics for the
MCAT. These are average scores, with a maximum of 15 possible on the
Physical Sciences, Biological Sciences, and Verbal Reasoning. The
Writing Sample is scored on a scale of J to T (don?t ask me why). The
composite score is out of a maximum of 45. A score of 40 is phenomenal
and 35 is considered a great score.

Physical Sciences?Applicants:  9.0
Biological Sciences?Applicants:  9.3
Verbal Reasoning?Applicants:  8.8
Writing Sample?Applicants:  P
Composite?Applicants:  28.1

Physical Sciences?Matriculants:  9.9
Biological Sciences?Matriculants:  10.2
Verbal Reasoning?Matriculants:  9.5
Writing Sample?Matriculants:  P
Composite?Matriculants:  29.6

(http://www.aamc.org/data/facts/2003/2003mcatgpa.htm)

* * * * * Research Medical Schools:

(More from the AAMC website http://www.aamc.org/students/considering/gettingin.htm)
Here is a list of questions that the AAMC recommends asking yourself
as you consider selecting a medical school:

-Do I prefer a small or large school?

-Do I like a large class or a small class? 

-Am I interested in a career in research, in clinical practice in
academic medicine? While every school offers opportunities to prepare
for careers in all areas, the variety of curricular experience varies
from school to school. Try to select schools that fit best your career
goals.

-Which schools have a learning approach that emphasizes primary care,
patient education, prevention, and preparation for community practice?
What schools have a teaching approach that will work well for me?

-What kind of financial resources will I need to attend medical
school? What are the costs?

-What types of financial aid are available at the schools I am considering? 

-Are the schools in a location that meets my needs? 

-Are the schools connected to a university or are they free-standing institutions?

-When should I be ready to apply to medical school?

-Most people, about 90 percent, apply to medical school at the end of
their junior year in college and begin their medical studies after
graduation. Others finish college and work for several years before
beginning their medical education. Still others participate in "early
admissions" or other collaborative programs between undergraduate
colleges and medical schools, through which students with a
demonstrated level of maturity and academic achievement can proceed to
medical education at an accelerated pace.
 
There is an excellent list of questions compiled by students that is
called ?31 Questions I wish I?d asked.? You can see it here:
http://www.aamc.org/students/applying/about/31questions.htm

The AAMC also publishes Medical School Admissions Requirements (MSAR),
a valuable resource in your specific research.
http://services.aamc.org/AAMC_Store/itemDetail.cfm?store=PUBLICATIONS&item=ARA3&category=

You should also consider contacting medical schools you are interested
in attending and speaking with someone in the admissions office. They
should be able to supply you with important information such as:

-What are the key deadlines for applications?
-What information is most important in the application?
-Are there student or faculty liaisons that are available to answer
questions regarding the school, its academics, extracurricular
programs, etc.?
-What are the statistics regarding number of applications, class size,
minorities and women, out-of-state versus in-state applicants, etc.?
-Any other questions that you can think of. Now is a good time to get
good information to prevent you from wasting time later on a school
that will not fit your needs.

* * * * * Application Preparation:

Much of the following information comes from the AAMC website
(http://www.aamc.org/students/considering/gettingin.htm).

There is also information available at the AAMC Application Service
(AAMCAS) website
http://www.aamc.org/students/amcas/aboutamcas.htm

(Please note that this information applies only to allopathic (MD)
schools and does NOT address osteopathic (DO) schools.)

Your Premedical advisor will help guide you through the process of
applying to medical school. Many universities have official Premedical
Committees that help applicants prepare for applications. This
assistance may come in the form of transcript reviews, practice
interviews, and individual guidance from the Committee Chair. My
husband?s committee had him complete an application similar to the
official AAMCAS application, including listing out his courses and
grades. He also had to write a personal statement, which will be
requested at every stage of the application process. They also
collected all of his letters of recommendation into this packet so
that there was one official set for future use. His information was
then sent to a committee member who interviewed him and wrote an
evaluation. He was also interviewed by the Premedical Advisor who
completed the evaluation process. The advisor then wrote a summary of
all of this information for submission to medical schools. This is an
excellent way to accumulate all of the application materials into one
document making review by an admissions committee more feasible.

Many universities use similar processes. Be sure to consult with your
Premedical Advisor to determine which procedures if any are in place
to assist you with your application.

- - - - - - - - - - - - - - - - - - - -

Central Application:

You should also obtain the AAMCAS packet from your premedical advisor.
This will include all of the instructions and materials necessary to
apply to most of the medical schools in the nation. You submit this
packet to the central application service, and the service will send
your information to as many schools as you designate. There is no
maximum number of schools allowed, but cost will be prohibitive. As I
mentioned before, it costs $150 to send an application to one school,
and $30 for each additional school. So applying to 10 schools, a
reasonable number, would cost $420. The deadlines for submission vary
from school to school, with the earliest deadlines being October 15.
For a complete list of school deadlines, see
http://www.aamc.org/students/amcas/deadlines.htm. The rule here is
?apply early.? Also, keep in mind, it would be wise to only apply to
programs in which you truly are interested. This will save you a lot
of time and money and ensure that the medical school that you attend
is one that suits you.

- - - - - - - - - - - - - - - - - - - - -

Letters of Recommendation

You should also be accumulating Letters of Recommendation during the
Junior Year (if you haven?t already). Collect as many as you can, as
you can always choose which letters to submit. It would be wise to
collect them from a variety of sources: science professors, other
professors, physicians with whom you have worked (including an
Osteopathic Physician if you are applying to Osteopathic Medical
School), anyone connected with your volunteer experiences, etc.

There are a number of sites with advice regarding these letters. 

From The Princeton Review?Medical School Recommendations
http://www.princetonreview.com/medical/apply/articles/other/recommendations.asp
(And it should be noted that this article is an excerpt from ?Complete
Book of Medical Schools,? 2002 Edition, by Malaika Stoll.)

?Letters of recommendation for medical school work in much the same
way as any other such letters; you will have much better luck if you
approach your potential recommender with a copy of your resume,
transcript, and personal statement. Try to make an appointment to
speak to potential recommenders in person to explain to them about the
various schools to which you are applying, and to make your case.?

Continuing on regarding whom to ask:
?Even if you have been out of school for awhile, you should try to get
at least one letter from a former professor. Medical schools are
interested in your character, your desire to be a physician, your
academic preparedness, and your intellectual ability. Although most
employers could attest to some of those qualities, you will probably
need a letter from a professor that discusses your academic abilities.
As an undergrad or in your post-bacc program, try to build
relationships with faculty members so they can write something
meaningful about you. Don't ask for a letter from someone famous
unless they know you pretty well. Name dropping is not considered to
be particularly attractive in a prospective medical student.
Although many returning adults feel awkward approaching professors
they might not have spoken with in several years, most are pleasantly
surprised to discover that, for the most part, professors do tend to
remember their students, and most are happy to write them letters of
recommendation. Both current and former students should also consider
asking for letters from doctors with whom they have worked or
volunteered--and remember that if you're applying to osteopathic
school, you have to have a letter from a D.O.?
More good advice on obtaining good letters of recommendation can be found here:

http://gradschool.about.com/gi/dynamic/offsite.htm?site=http%3A%2F%2Fwww.socialpsychology.org%2Frectips.htm

Though the information here seems geared toward people applying to
graduate school (hence the GRE references), many of the tips also
apply well to medical school applicants.

More information can be found here (this link is through about.com and
comes from Berkley):

http://gradschool.about.com/gi/dynamic/offsite.htm?site=http%3A%2F%2Fcareer.berkeley.edu%2FGrad%2FGradLetter.stm

Again, though the information here seems geared toward people applying
to graduate school (hence the GRE references), many of the tips also
apply well to medical school applicants.

8 great tips from the About.com site:
http://gradschool.about.com/gi/dynamic/offsite.htm?site=http%3A%2F%2Fpsych.hanover.edu%2Fhandbook%2Frec_letters2.html


Here are some interesting points about professors? pet peeves to help
give some idea of what NOT to do:

http://gradschool.about.com/gi/dynamic/offsite.htm?site=http%3A%2F%2Fpsych.hanover.edu%2Fhandbook%2Fpeeves2.html
?Professors? Pet Peeves: How to Receive a Less Than Enthusiastic
Letter of Recommendation?
William W. Nish, Georgia College

- - - - - - - - - - - - - - - - - - - -

Here is an article by George Ruiz, M.D. discussing advice for getting
into medical school:

From the September 2002 (volume 7, number 3) newsletter for ?In touch
with Biomedical Science Careers Program?
http://www.bscp.org/Upload/Files/September%202002.pdf

?Getting Into Medical School: The Honest-to-Goodness Truth
By George Ruiz, M.D.

I?m listing his 10 main points, but I highly encourage you to read the
entire article as his advice and insight is very valuable. This is a
very good article.

1. Choose a major you are genuinely interested in. 
2. Cultivate interests outside of the classroom. 
3. Study for the MCATs. 
4. Choose your letter writers wisely. 
5. Apply to a broad range of schools to maximize your chances.
6. Do not be the applicant as well as the selection committee! 
7. BE ON TIME. 
8. Make your personal statement personal. 
9. Do not be intimidated at the interview. 
10. Never be discouraged. 

= = = = = = = = = = = = = = = = = = = =

Senior Year

* * * * * Major:

The only advice now is to finish it up and finish it well. Make sure
that you?ve completed or will complete all of your requirements by
graduation time. This includes required courses, reading lists,
thesis, etc. Don?t let a little work within your major hold up your
medical school admission

* * * * * Courses:

First of all, be sure to finish your major coursework. Next, ensure
that you?ve completed all of your GE courses as well. More than one
student has had to scramble at the end to finish up GEs?don?t be one
of them!

If you find yourself with time on your hands, with nothing else to do,
I make the following suggestions:

-Take, you guessed it, an ?enrichment? course. This is your last
chance. Use it wisely

-If you are so inclined, you may consider taking ?extra? science
courses. Some students find this helpful in preparation for the first
year of medical school. Some suggestions I?ve heard include
Microbiology, Immunology, and Biochemistry. These are courses where a
little extra exposure might make the experience more ?enjoyable?
later.

* * * * * Extracurricular Activities:

As with just about everything else, this is your last chance to be
involved in university extracurricular activities. They will continue
to enrich your life, and they will continue to give you experiences to
speak about during your interviews, which will occur during this year.

* * * * * Employment:

I?d keep working if I were you. You will probably never raise enough
money to pay for the four-year bill due at your medical school, but
every little bit helps. This is also going to likely be the last
chance you have to work for any appreciable amount of time before
medical school starts and especially during medical school.

* * * * * Application Process:

Before filling out your applications, it is useful to understand some
of the ?key factors? admissions committees use in determining which
students they will accept into their programs. Here is a list of some
of those factors from the AAMC website

?The key factors affecting acceptance to medical school include the following: 
-Successful completion of required undergraduate courses 
-Grade point average 
-Performance on the Medical College Admission Test (MCAT) 
-Extracurricular activities - especially those reflecting public or
health-related service, volunteer work, and other evidence of your
initiative
-Letters of recommendation from undergraduate health professions
advisors and faculty members as well as physicians and other members
of the health professions, community leaders, and other individuals
who have employed you or supervised your volunteer experience
-Interviews with medical school admissions committees. Unlike
colleges, which hold interviews early in the application process,
medical schools arrange them near the end. As they narrow their
selection of candidates, most medical schools invite the most
promising applicants to interview with faculty and other members of
the admissions committee.?

Technically, you don?t have to have a bachelor?s degree to get into
medical school (although most applicants do and many have, so to not
have a degree puts you at a disadvantage comparatively), so long as
you complete your prerequisite courses. No matter what your major, you
must complete the premedical coursework necessary to apply and get
into medical school. You can verify that you?ve completed the
necessary requirements by checking with the schools to which you would
like to apply.

It is also useful to have hands on experience to ?beef up the résumé.?
Many applicants have worked or volunteered in medical fields and know
firsthand what it is like to be involved in medicine or
medicine-related experiences. Any experience you have that sets you
apart from the rest most definitely gives you a competitive edge.

Writing a good personal essay is important. These essays generally
have to do with why one wants to be a doctor and things of that
nature. Also, having impressive letters of recommendation is an
important part of the application process.

Having worked in admissions, I can give you some tips. The caliber of
applicants to medical school is quite impressive. At the university
where I worked, it seemed that each year the applicants got better and
better. Many people do not get into medical school the first time they
apply. In my opinion, the ?weeding out? process with regards to
becoming a doctor happens more in the application process than in the
schooling itself. Once you?re in, you?re likely to stay in. The
applicants that are trying to get in for a second and third time show
true determination, as they gain more experience and try again. I saw
students studying medicine fresh out of college. I saw students who
took time off between college and medical school to work and gain more
life experience. I saw students in their 50?s at the University where
I worked. I saw lawyers who were studying to become doctors. I saw
businessmen who were studying to become doctors. I saw RNs who were
studying to become doctors. The class demographics are certainly
diverse! Again, anything you can do to set yourself apart is a big
plus.

Depending on the kind of school to which you apply (allopathic?MD or
osteopathic?DO), emphasizing different aspects of your application is
beneficial. Being well rounded is good no matter what you do, but
osteopathic schools, due to their philosophy, tend to have a more
holistic approach even in screening medical school applicants. For a
look at the difference between MDs and DOs, please refer to:
http://www.aoa-net.org/Consumers/whatdo01.htm.

After applying to medical school, applicants anxiously await an
invitation to submit secondary applications. The universities that
invite applicants to submit secondary applications generally want more
specific information about the applicant and they also usually charge
more fees (yes, fees are a part of the whole process!). The applicant
submits ?secondaries? (more lingo for you!) and eagerly hopes for an
invitation to interview. Get these secondaries in early so that you
can arrange your interview day sooner. Many colleges have ?rolling
admissions? meaning that they will accept students as they interview
them. Interviewing early will give you a greater chance at getting in.
Some colleges wait until all of their interviews are completed to make
their decisions. It is still in your best interest to interview early
so there is less of a chance of having a conflict of interest or
difficulty fitting into their schedules.

* * * * * Interviews:

Interview day can be stressful or relaxed, depending on the approach
of the interviewing school, although most applicants are nervous
either way. Interviews are usually done in either a series of
individual interviews or by a panel. My husband has sat in on a member
of the interviewing panel on a number of these such interviews and
sees quite a range of questions, from testing an applicant?s medical,
scientific knowledge to asking what the applicant?s favorite book or
hobby is. Try to find other students who have interviewed at the
schools you will be visiting and ask them about their interview days.
Their experiences are usually good indicators as to what you can
expect. A good place to check is The Student Doctor Network,
www.studentdoctor.net. This is an online forum where students discuss
issues related to pre-health and health professions education (For
information on interviews visit
http://www.studentdoctor.net/interview/index.asp). Many students will
post information about their interviews, including interesting and
difficult questions, and an overall impression of the interview day
and school in general. Another good source of information is the
website for each school where you will interview. This will allow you
to ask intelligent questions and answer questions about your reasoning
for applying at that school.

http://chemistry.about.com/library/weekly/aa013102a.htm
About.com's medical school interview advice

http://www.mymedline.com/premed/index.php3
A Premed Advisor with step-by-step assistance for the interview process

* * * * * Matriculation:

Once a person is accepted to medical school, yet more fees are
collected in order to secure a spot. Because entrance to medical
school is quite competitive, this is often an investment as many
applicants apply to numerous medical schools and put money down to
secure spots at any universities that accept them (there are stringent
deadlines for fees) and then decide which university to attend. It is
possible to put a sizeable amount of money down at one school (as a
back up) while you wait to hear back from other schools. You may or
may not get back some or all of that money. Again, it is an investment
and part of this whole process.

You will be given a packet of matriculation information which will
give you instructions regarding deposits, immunizations, start dates,
etc. Be sure to review this information carefully and meet all of the
deadlines required. Contact a representative in the admissions office
as necessary to clarify anything along the way.

Clearly, applying to medical school is an involved and
somewhat-daunting process, but it is also a very exciting and
(hopefully) rewarding one.

* * * * * Graduation:

Sometime early in your Senior year, consult with your academic advisor
within your major and make sure that you?ve completed all of the
necessary requirements for graduation. There will be a variety of
deadlines depending on your university?s particular procedures. You
will probably have to register to graduate at some office on campus.
Your advisor can help you determine where. Then you should receive
information with the various deadlines and requirements. Don?t miss
them! You?ve worked too hard to let late reservation of a cap and gown
stop you from graduating!

Enjoy the feeling of being a college grad, and do something fun! You?ve earned it!

* * * * * Relaxation: 

You?ve worked hard, very hard, and you deserve to take some time for
yourself. Plan to do something that you really enjoy during the time
between Graduation and Matriculation. Most medical students who don?t
take time to relax really regret not doing so. Medical school is a
long process and it helps to go into it refreshed.

= = = = = = = = = = = = = = = = = = = =

The good news is that from here on out, things are more cut and dry. I
didn?t say stress-free, but more cut and dry.

An Overview of Medical School 

Medical School is 4 years long. I?m tempted to say
?looooooooooonnnnnnnngggggg? but I won?t. Remember, it?s very fresh in
my mind! For the first two years, students are immersed in intense
academic study, including lots of lectures and lots of testing! The
amount of material covered and the credits gained from the classes is
much like completing 4 years of college in 2 years. It?s A LOT of
information! Many colleges adhere to a pass-fail system; others use
letter grades.

Every medical school differs a bit in curriculum, but as you?ve asked
for specifics, I?ll go ahead and list an outline of curriculum that
I?ve found online to give you an idea. Most medical schools follow
something similar with subtle differences, so this should give you a
good idea.

Some schools use a semester system and some use a term/quarter system.
There are pros and cons to both and finding out what suits you best is
a good idea. A good way to get feedback on this is to talk to other
students. Again, studentdoctor.net is a valuable resource.

The following is from ?How Stuff Works? and refers specifically to
becoming a doctor (it is written by an M.D.) and it is another stellar
resource that I highly recommend!

http://people.howstuffworks.com/becoming-a-doctor9.htm

In part, the site states:
?The 1st year of medical school consists of basic sciences which
educate you about normal body functions. Courses include:
·	Anatomy - the science of the structure of the body and the relations
of its parts (especially those that can be seen with the naked eye).
This course is taught by lecture and dissection of a human cadaver
·	Physiology - the science of the function of the body and its parts
(and the chemical and physical steps involved)
·	Histology - the science of the cells and tissue on the microscopic
level (microscopic anatomy)
·	Biochemistry - the science of the chemistry of the cells, tissue,
and organs of the body
·	Embryology - the science of the development of a human from its most
immature form
·	Neuroanatomy - the anatomy of the nervous system ?
The site goes on to discuss the second year curriculum:
?In the 2nd year of medical more emphasis is placed on disease and its
treatment. Courses in the 2nd year include:
·	Pathology - the science of disease, especially the changes in
structure and function causing or caused by disease
·	Pharmacology - the science of medications 
·	Microbiology - the science of microorganisms such as bacteria,
viruses, and fungi
·	Immunology - the science of how humans fight disease 
During the 2nd year medical students also take courses that introduce
them to taking a medical history and performing a physical exam.?

The 3rd and 4th years of medical school are made up of some didactics,
but mostly clinical rotations. Students rotate through a number of
specialties for two years. The rotations are generally 4 weeks long,
though they can vary from about 2 weeks to 6 or more weeks. There are
certain core rotations that all students are expected to complete,
such as Family Medicine, Surgery, Obstetrics, Pediatrics, and Internal
Medicine. Some schools may also include such rotations as Emergency
Medicine and Neurology. There are usually some elective rotations that
allow a student to emphasize in different fields to see what they like
or to gain more experience for applying to residency.

Some people go through medical school knowing exactly what they want
to specialize in and others change their minds all through medical
school and even beyond. Peoples' experiences vary greatly depending on
their approach. As I?ve mentioned before, my husband graduated medical
school this past June (we are in residency) and has always been
committed to being a family doctor, so it was fairly cut and dry
during medical school to know what kind of electives to take in order
to gain more knowledge in specific areas that interest him rather than
taking different electives to try to figure out what to specialize in
(as MANY of his classmates did).

Okay, board exams. Students take board exams in medical school as
well. These are taken after 2 years and again after 4 years (and again
in and after residency). These exams test medical knowledge and
skills.

http://people.howstuffworks.com/becoming-a-doctor10.htm

With regard to the Boards, the site states:
?At the end of 2nd year all medical students take the first part of
the United States Medical Licensing Examination (USMLE-1),
administered by the National Board of Medical Examiners (NBME). The
purpose of the NBME is to prepare and administer a test that states
feel confident in using to determine medical competency when deciding
to issue a license to practice medicine. There is no national license
to practice medicine. Each state has its own rules and issues its own
license.
The USMLE is a three-part exam: 
·	Part 1 is taken in the 2nd year of medical school and tests your
knowledge in the basic medical sciences.
·	Part 2 is taken in the 4th year of medical school and tests whether
you can apply your medical knowledge in providing patient care under
supervision.
·	Part 3 is taken in the first year of residency (internship) and
tests your ability to apply your medical knowledge in providing
unsupervised medical care.
These are very difficult exams. However, over 90% of students from
U.S. M.D. schools pass these exams (see www.usmle.org). ?

And to reiterate the rotation curriculum for the 3rd and 4th years again:
?In the 3rd year, medical students begin rotating through each of the
specialties of medicine, such as Internal Medicine, Surgery,
Pediatrics, Obstetrics and Gynecology, Psychiatry, Neurology,
Radiology, Emergency Medicine, Family Medicine, etc. In the 4th year
more clinical rotations with a higher level of responsibility are
completed, in addition to electives. ?
Oh yes, and let?s not forget a big aspect of medical school for
most?DEBT. Going to medical school is a full-time endeavor and it is
nearly impossible to go to medical school and work full-time
simultaneously. (I personally don?t know anyone who has done this.) It
can be a very, very, expensive proposition.

= = = = = = = = = = = = = = = = = = = =

Other Things to Do in Medical School

- - - - - - - - - - - - - - - - - - - - - -

First Year (MS-I, or ?Medical Student Year I?)

* * * * * Extracurricular Activities

Just like applying to medical school, applying to residency programs
is a competitive process. Being involved in extracurricular activities
will play an important role (again) in that process. There will be
many ?interest groups? or clubs related to different medical
specialties. This is a great way to explore different careers if you
are unsure of your ultimate plans. These clubs will often arrange
workshops in clinical skills (such as suture workshops, casting
clinics, etc.) and will have many guest lectures relating to their
fields. This is a great way to build camaraderie among classmates and
begin networking in the field.

You should also look into organizations that provide community
service. Some will be involved in community health clinics and other
health-related fields. This is a good way to give back to the
community as well as hone your medical skills. My husband participated
in a number of sports physical days in addition to health fairs
providing free physicals to the underserved population in our area.
They were all great experiences for him. As always, balance out your
involvement in these groups with your studies and your personal and
family time.

* * * * * Board Exam Preparation

As I explained above, your first Board Exams will be following your
MS-II year. It might be early to begin vigorous preparation at this
stage, especially with a demanding MS-I coarse load, but you can
always be researching the best supplemental texts for Board
preparation. You can also ask other medical students about their
experiences on Board exams. You will find that medical school tends to
be a very collaborative process.

* * * * * Finances

Unless you are going to a state school (or have rich and generous
family) you will probably not be able to completely pay for your
tuition and living expenses. Also, there is rarely time to be involved
in part-time employment. For this reason you should get to know your
financial aid office. They will be instrumental in guiding you through
the finances of a medical education. They will also provide you with
information regarding federal work study programs that allow you to
make a nominal amount of money in exchange for working on campus doing
various jobs, such as filing, lab work, etc.

* * * * * Residency Research/Application Preparation

It is never too early to begin preliminary research into possible
residency programs. Here is a useful collection of websites related to
various residency programs:

http://www.siumed.edu/lib/ref/StudentPages/IM/finding.html

Just like with medical school applications, keep you résumé current.
This will save you a lot of time later. You will probably not have
many significant contacts at this time, but you may start collecting
letters of recommendation now according to some of the guidelines I?ve
suggested earlier. Contact someone in the Clinical Education
department at your school to determine how to collect them.

- - - - - - - - - - - - - - - - - - - -

Second Year (MS-II)

* * * * * Extracurricular Activities

Continue doing what you?re doing. Enjoy and don?t over extend
yourself. Focus on activities that you truly enjoy.

* * * * * Board Exam Preparation

With board exams happening at the end of this year, you should really
begin to concentrate on specific board exam preparations. You can use
books, workbooks, study groups with other students, or whatever helps
you study and prepare the most. Talk to students who have recently
taken the board exams and ask which review courses and review texts
have helped them most. Remember to pace yourself so that the exams
don?t sneak up on you. The MS-II curriculum is very demanding, so you
don?t want to get behind on your coursework while trying to study.
These courses should be ?board review? in and of themselves. Balance.
Balance. Balance.

Be in contact with you clinical education department as they can help
you do what you need to do to meet deadlines and prepare for taking
the exam.

* * * * * Finances

Keep borrowing (again, unless you?re very rich) and stay in contact
with your financial aid department to have the proper forms and meet
the proper deadlines.

* * * * * Residency Research/Application Preparation

Continue to research. If there is a program in which you?re
particularly interested, now is a good time to consider whether or not
you?d like to rotate there for 2 reasons?you can check the program out
firsthand and they can also see that you?re sincerely interested and
dedicated. You don?t HAVE to do this, but in some cases, it may help.

Applications to residency begin between the 3rd and 4th year of
residency, so the earlier you can hone in on a specialty, the easier
it will be to complete the applications (which you can obtain online).
Again, your clinical education department will have the information
that will guide you through the entire process. Also, you will be
going through this along with your classmates and there is usually a
great deal of helpful information sharing.

We got our hands on a big book of all the residency programs in the
country and narrowed down the list according to our own priorities
(including location, schedule, philosophy, etc.) and kept referring to
the book and doing lots of online researching.

- - - - - - - - - - - - - - - - - - - -

Third Year (MS-III)

* * * * * Extracurricular Activities

Continue doing what you?re doing. You will likely have little time to
participate in much, due to a demanding schedule, but continue
enjoying the things you truly like to do.

* * * * * Board Exam Preparation

There?s not much to do during the third year. In fact, simply
participating in your rotations will be board preparation enough in
many cases. Read additional books as you feel necessary.

* * * * * Finances

Keep borrowing (have you won the lottery, yet?)!

* * * * * Residency Research/Application Preparation

Applications begin at the end of the 3rd year, so ideally, by now, you
should have a fairly good idea of both specialty and programs in which
you are interested. Continue doing that which I?ve listed under the
second year preparations. As you get closer to deadlines, it will
become very cut and dry what you have to do as you meet certain
program deadlines. Remember, interviews are next year, so research
your programs well. Talk with 4th year students and hear about their
experiences and insights.

- - - - - - - - - - - - - - - - - - - -

Fourth Year (MS-IV)

* * * * * Extracurricular Activities

You will likely have even less time to participate in much, due to a
demanding schedule and residency applications and interviews, but
continue enjoying the things you truly like to do.

* * * * * Board Exam Preparation

Again, simply participating in your rotations will be board
preparation enough in many cases. Read additional books as you feel
necessary. There?s an old adage that goes ?study 2 months for step 1,
2 weeks for step 2, and bring a #2 pencil to step 3.? My husband finds
that to be fairly accurate. Talk with other students and also with
your clinical education department to keep up on ways to best prepare.

* * * * * Finances

Make sure that you?ve assessed your finances well, keeping in mind
that you will spend more money in the 4th year to apply to programs,
including travel. Depending on the kind and number of programs, this
can be rather costly. Be wise. Talk with others. Don?t underestimate,
yet don?t go too crazy. Balance. Balance. Balance.

* * * * * Residency Research/Application Preparation

Your applications go in at the beginning of this year, so make sure
that you?ve narrowed down your choice of specialty and programs by
this time (which you?ll be able to better choose due to learning,
rotations, talking with others, personal experience, etc.) One good
piece of advice is don?t apply anywhere that you wouldn?t want to end
up going. You don?t have time or money to waste on programs in which
you?re not truly interested.

Enjoy Graduation! You?ll probably be exhausted and more than ready for it!!

- - - - - - - - - - - - - - - - - - - -

Applying to Residency

After graduation, new doctors train in residency programs. Applying to
residency is a very involved (and somewhat costly) process as well and
I encourage you to check out the specifics of matching and scrambling
and interviewing at the following site (it?s pretty interesting, I
assure you). The best site for learning about the match (as it can be
quite complicated) is the website for the national resident matching
program found here: http://www.nrmp.org/. As you go through medical
school and interact with other students, you will be well aware of
this process and better understand how it works.

A brief overview from the website is as follows:

?Candidates must apply to the programs of their choice using the
method accepted by the program. Most programs participate in the
Electronic Residency Application Service (ERAS). A few programs may
use their own application or the Universal Application for Residency.
Applicants also are responsible for ensuring that they meet all
program prerequisites and institutional policies regarding eligibility
for appointment to a residency position prior to ranking a program
through the NRMP. In particular, non-US citizens should inquire about
visas accepted by the institution.

ERAS is the Electronic Residency Application Service developed by the
Association of American Medical Colleges to transmit via the Internet
residency applications, personal statements, recommendation letters,
deans' letters, transcripts, and other supporting credentials from
medical schools to residency program directors.?

The site goes on to explain:

?Candidates should carefully review the eligibility requirements when
applying to residency programs. For example, one consideration for
applicants living abroad is that many programs require on-site
interviews. Applications should be submitted as early as possible in
order to accommodate deadlines for supporting documents and interview
schedules. Late applications may not be considered. Do not send any
program applications or the Universal Application for Residency to the
NRMP.

Candidates also should carefully review application materials to
determine if the program requires applicants to enroll in the NRMP to
obtain residency positions.?

More sound advice from the site:

?Programs are expected to provide complete and accurate information to
interviewees, including a sample contract and all institutional
policies regarding eligibility for appointment to a residency
position. This information must be communicated to interviewees in
writing prior to the rank order list certification deadline. It is
recommended that each program obtain a signed acknowledgement of such
communication from each applicant who interviews with the program.

The ranking of an applicant by a program or of a program by an
applicant establishes a commitment between them should they be
matched. Failure to honor that commitment is a violation of the Match
Participation Agreement, and violators may suffer penalties as
outlined in Section 7.0 of The Agreement and in the NRMP's Violations
Policy. However, appointments are dependent upon applicants meeting
all eligibility and appointment requirements in effect at the time
rank order lists are submitted. Institutional policies take precedence
over the results of the Match.

The Match Participation Agreement does not preclude one party from
expressing a high level of interest in the other; however, any attempt
by either party to request such information is a violation of the
Match Participation Agreement. Statements implying a commitment should
be avoided. Rank order lists take precedence over verbal commitments.?

As you can see, it is quite involved, but it is navigable with the
assistance of your clinical education department.

To give you an idea of residency time frames, general practice,
commonly referred to as ?family practice? these days, is a 3-year
residency (training) program. This is one of the shorter residency
programs compared to most specialties. For example, residency programs
in specialties like orthopedic surgery are 5 years and specialties
like cardio thoracic surgery and neurosurgery are even longer. Right
now, family practice is one of the less competitive programs to get
into (there are many speculative reasons for this including lower pay,
participation in OB, not specializing, not considered very
prestigious, etc.). Currently, some of the more competitive
specialties (and by the way, these often change over the years)
include radiology, emergency medicine, and orthopedic surgery. In my
opinion, a person should do what he or she wants to do and pursue
fulfillment.

During the 4th year of medical school, you will have Match Day. You
will find out whether or not you matched and a couple of days later
(oh, the stress!), you will find out where (if you matched) and if you
didn?t match, you will have to scramble (an appropriate name) for a
spot. You will find a way to be in some program somewhere. You and
your program will communicate and sign contracts and make it all
official. If you?ll be moving, start packing. That?s my personal
advice and believe me, it?s good advice.

- - - - - - - - - - - - - - - - - - - -

Here is some advice from George Ruiz, M.D. regarding residency
applications. Since his prior article was so good, I decided to
include this one here.

 From the January 2003 (volume 8, number 1) newsletter for ?In touch
with Biomedical Science Careers Program?
http://www.bscp.org/Upload/Files/January%202003.pdf

?Applying for Residency
By George Ruiz, M.D. 

1. Decide what you think you want to do. 
2. Plan your fourth year wisely. 
3. Choose your letter writers carefully. 
4. Research: parts A and B. (Basically, conduct research and also
research the programs).
5. Prepare your curriculum vitae. 
6. Prepare your personal statement. 
7. Be on time. 
8. Never be the applicant as well as the selection committee. 
9. Never be discouraged. 
10. Do not be intimidated by the interview.

= = = = = = = = = = = = = = = = = = = =

Residency 

The first year of residency is called the internship year. Interns
work many, many hospital (and some clinic) hours, take a lot of call,
and often do a lot of ?scut work? (residency lingo for menial tasks
that others don?t always want to do and that often are given to the
interns to do). There are all kinds of new laws in place (started this
year!) to make sure that work hours are more humane than they?ve been
in the past. So, interns now can get a little more sleep than interns
in the past.

Technically, one could complete the internship year and then begin
working as a physician in settings such as urgent care clinics.
However, to become board-certified, completion of a full residency
program is necessary. Earlier, when I referred to a range of ?5-7
years,? I was alluding to this. During your internship year, or
possibly during your second year, you will need to complete Step III
of your board exams. This exam is traditionally fairly straight
forward since you are practicing the information on a daily basis.
Your program will ensure that you get the necessary information
regarding deadlines and content. Many programs will incorporate board
review into their formal educational component to maximize the
interns? time.

Together with the internship year, the other years (again, for family
practice, that?s 3 years and for other specialties, that often and
usually includes more) make up the residency. After residency, doctors
can elect to do additional fellowships in specialized areas. For
example, for family practice, there are specialties in areas such as
obstetrics, sports medicine, and geriatrics. Internal Medicine
fellowships include specialties such as Cardiology, Endocrinology,
Infectious Disease, Gastroenterology, Rheumatology and others. The
surgical specialties also have fellowship programs, so as you can see,
you could study nearly indefinitely if you wanted to.

A nice thing about residency is that you get paid. But not much. We?ve
heard a positive spin on it, as friends of ours refer to it as an
$80,000 raise (rather than borrowing 40 thousand a year?at some
schools?you make about 30 or 40 thousand a year)! Now, that?s
perspective! One of our favorite lines from a popular sitcom called
?Scrubs? (a show about resident life) goes something to the effect of:
?The average resident is $125,000 in debt and makes about as much as a
waiter.? Though residencies are paying more than they have in the
past, for the hours you put in, it is like making a waiter?s salary
per hour.

= = = = = = = = = = = = = = = = = = = =

Working as a Doctor (post residency)

Doctors continually learn and keep up to date by completing yearly CME
(continuing medical education) units. And this is about where you
begin to earn a generous salary. To give you an idea, the average
salary for family physicians is about $136,636 according to the site
Salary.com. That and more information can be found at
http://secure.salary.com/jobvaluationreport/docs/jobvaluationreport/joblisthtmls/jvrjob_HC07000052.html.

Also, depending on the specialty, there will be a specific process for
becoming ?board certified.? This will most likely involve an
examination and certain work experience requirements. You will need to
keep up on these requirements as you progress through your specific
residency.

= = = = = = = = = = = = = = = = = = = =

Some great resource sites (some may repeat from earlier):

AMA (American Medical Association)?Becoming an MD
http://www.ama-assn.org/ama/pub/category/2320.html

AACOM (American Association of Colleges of Osteopathic Medicine)
http://www.aacom.org/

AAMC: Considering a career in medicine
http://www.aamc.org/students/considering/start.htm

Medical Careers for High School Students
http://www.bestpremed.com/hs.php

Student Doctor.net
www.studentdoctor.net

How Stuff Works??How Becoming a Doctor Works?
http://people.howstuffworks.com/becoming-a-doctor1.htm

Association of American Medical Colleges
www.aamc.org

American Association of Osteopathic Colleges of Medicine
http://www.aacom.org/

Information on AACOM Application Service
https://aacomas.aacom.org/

Information for High School Students considering a career in medicine
http://www.bestpremed.com/hs.php

Beginning information for prospective medical students
http://www.aamc.org/students/considering/start.htm

?So you?re going to medical school? (Canada)
McMaster University
http://careers.mcmaster.ca/pathfinders/medschool.htm

Medical School Admissions: Getting into Medical School
http://www.premedguide.com/gettingin.html

= = = = = = = = = = = = = = = = = = = =

Phew. I?m tired! It?s been interesting to recap all of this for you.
In some ways, it?s made me think, ?WOW! We?ve done (and are doing) all
this!! Wow!? Maybe it?s good that I didn?t read an answer like this
before it all (it might have scared me away!), so hopefully your
daughter isn?t too overwhelmed by this and continues to be excited
about it!

Also, one of the things that I?ve been working on in the time that
I?ve had your question locked is contacting some admissions staff
friends and medical students and colleague doctors and asking them
what their advice to a junior in high school would be. As I hear back,
I?d be more than happy to post the information here for you. Would you
like that?

The good thing here is that we?ve been through it/we?re going through
it, so we can help you very much, yet at the same time we might take
for granted some bit of information about the process, so please
continue to ask for clarification as needed so that we may continue to
help you! It has been a pleasure to answer your question. I wish you
and your daughter much success and again, I applaud what you?re doing
in obtaining this information at this stage of her education! Should
you need further clarification, please let me know how I can help. 
Happy Holidays!

Sincerely,
Boquinha-ga

Search Strategy:
Personal Experience
Getting feedback from others with personal experience
Using sites that we?re already aware of
?Going to Medical School?
?Getting into Medical School?
?Premedical curriculum?
?George Ruiz M.D. getting into residency?
?high school medical school?
?letters of recommendation medical school? 
?AOA getting into medical school?
?MCAT?
?Becoming a doctor?
?Medical school curriculum?
?Medical school interviews?

Request for Answer Clarification by jmswilsn-ga on 24 Dec 2003 06:53 PST
Hello boquinha,

You are so great! Thank you thank you and thank you again for all of
this information. To be honest, I'm still going through it and slowly
trying to digest it with my daughter. My apologies for not answering
or commenting before, Google has temporarily disabled my posting
rights because of some billing issues and did not re-enable them in
the promised time (that same day it was supposed to be.)

I will request clarification (if any) and post your 5 star (actually
100+ star) rating along with a nice, juicy bonus as soon as possible.

Thank you again!

Clarification of Answer by boquinha-ga on 24 Dec 2003 09:23 PST
Hello!

I'm very happy to hear from you! I've been wondering and hoping that
my answer has been helpful to you and your daughter. I'm sorry to hear
that you are having problems posting--I'm sure that that can be very
frustrating. Yes, please do read through it, digest it, and let me
know of any questions you may have. I've heard back from a couple of
people about "advice they would give to a junior in high school who is
contemplating medical school" and if you're interested, I'd be happy
to post that for you. I hope to hear back from more people as
well--I'm guessing that the delay in hearing back from some of those
people is due to the holidays. Happy reading to you both and please
let me know of any more help that I can give you! Merry Christmas Eve!

Sincerely,
Boquinha-ga

Clarification of Answer by boquinha-ga on 27 Dec 2003 14:03 PST
Hello again!

As I mentioned earlier, I have asked over 10 people (doctors,
family/support of doctors, people in medical school, people applying
to medical school, medical school admissions staff, etc.) what advice
they would give to a junior in high school who is contemplating going
to medical school. I have received 2 responses so far and anticipate
more as I suspect the delay in hearing from some is due to the
holidays. I?ll let you know what I?ve got so far and continue to post
as I hear more, as I know that you and your daughter are reading
through all of this and digesting it. Hopefully, these pieces of
advice can help add to and enhance the information that I?ve posted in
my answer to you.

The first piece of counsel comes from a doctor who recently graduated
medical school and is currently in residency. He basically says to aim
high and have lots of options. While he encourages getting good
grades, taking good classes, and getting into a good college, he
recognizes that that advice is good for anything you choose to do. So,
his biggest piece of advice is to get a broad range of life
experiences before applying to med school--travel, Peace Corps (if
that?s your thing), jobs, etc.

The second piece of counsel comes from a doctor and his wife and it?s
based on their discussions with a number of recent female graduates of
medical school. I?ll be very honest when I say that their advice is a
bit jaded by their interactions with others, but honestly so. Their
advice is, depending on your other life goals, don?t do it.
Particularly, if you want to get married and have a family, they
discourage the med school idea in favor of a more manageable
approach/degree such as Nurse Practitioner or Physician?s Assistant.
The woman who wrote me informed me that she has had several
conversations with women who recently graduated from medical school
who are now wishing that they would have thought things through a
little better?they either have children or would like to have children
and with that, they don?t feel that they are in a position to be the
kind of mother that they would like to be. My personal addition to
this is that this certainly does need to be considered when making a
huge decision such as this, but I also say that along with that very
important and serious consideration, aim high and follow your dreams.
Have no regrets. There?s something to be said for that and for having
the drive to do what?s right for you?somehow you find the energy to
pursue it and do it. Again, I reiterate that it?s important to balance
it all and be healthy about it (both for the doctor and the
family/support system) as it is an involved process and can be a
stressful pursuit. It?s certainly important to have priorities and to
live by them.

Again, I?ll keep you abreast of the information as I receive it. I
hope that this is helpful! Happy Holidays!

Sincerely,
Boquinha-ga

Clarification of Answer by boquinha-ga on 27 Dec 2003 21:04 PST
Hello!

I've got another response to share with you! As I haven't heard back
from you on this, I'll go ahead and continue to post them, but if
you'd rather I not, please let me know. I am assuming these are
helpful to you and your daughter. I hope they are!

This doctor gives the counsel to learn to balance now. He speaks of
having interests, hobbies, and outlets outside of medicine. It's
important to not focus in so narrowly on medicine that you lose the
ability to enjoy yourself outside of medicine. He mentions that he's
seen too many physicians whose only hobbies are to read medical
journals, attend medical conferences, etc. and then don't know how to
interact outside of that world. He goes on to say that balance looks
great to admissions committees and it preserves your sanity, too. I
think that's great advice.

Well, I hope to hear from you soon. I currently have another of your
questions locked (the NYC one) and I'm hard at work with that one, so
I especially hope that you're happy with my work! Again, good luck to
you and your daughter and happy holidays!

Sincerely,
Boquinha-ga

Clarification of Answer by boquinha-ga on 29 Dec 2003 18:35 PST
Hi, jmswilsn-ga!

Are you there? I'd really like to hear back from you on both this
question and the NYC one to make sure that you're okay with what
you're receiving and the information that I'm gathering. Could you
please confirm? Thank you so much! Hope you're having Happy Holidays!

Sincerely,
Boquinha-ga
jmswilsn-ga rated this answer:5 out of 5 stars and gave an additional tip of: $65.00
Thank you thank you thank you!

We took a great while to go through all the information, especially
with the holidays around. I have to say that I'm very glad that I got
specifically you to answer this question. Not only do you have the
intelligence, knowledge and background to answer this question, but
you are also very descriptive, analytical and know exactly what the
whole deal of getting a Medical Education is about.

I once again thank you from my very heart. My daughter and I, both
wish you the best of all.

Comments  
Subject: Re: Medical Doctor Career, Process of
From: boquinha-ga on 29 Dec 2003 21:01 PST
 
jmswilsn-ga,

Thank you so much for the 5-star rating, your kind words, and very
generous tip! What a wonderful holiday treat! I'm so grateful that I
can be a part of helping you and your daughter in this pursuit (and I
hope she hasn't been scared off by it all!) and if you have any more
questions about the process as you go through it, please let me know
how we can help! I will continue to post more information here as I
hear back from the people I've contacted for advice on this topic. I
hope that you and your daughter find that helpful as well. Again,
thank you! Every little bit helps us pay off those student loans (did
I mention those in the answer?) ;) Okay, now I'm off to do more
research to help make your daugther's trips to NYC fun, safe, and
eventful! Thank you and Happy Holidays!

Sincerely,
Boquinha-ga
Subject: Re: Medical Doctor Career, Process of
From: byrd-ga on 01 Jan 2004 15:39 PST
 
Hi jmswlsn,

Congratulations to your daughter on her commendable choice of career
and to both of you on your foresight in thinking of these challenges
now instead of, like most of us, waiting and just plowing through as
issues arise.  Congratulations too, to my esteemed colleague,
Boquinha-ga, for an exemplary answer.  Well done!  I just have one
small comment to add from personal experience that I hope may add a
little insight.

Many long years ago I too thought I would like to pursue medicine ...
naively I suppose since I'm really not suited to that, as I ultimately
realized. However, I was taking a lot of science courses in college
and found I absolutely *loved* chemistry, especially the little taste
of organic chemistry I had in the general course. So I blithely signed
up for organic chemistry only to discover to my dismay that it
required a heavy duty math background I simply didn't have.
Specifically, within the first week of classes the professor was
drawing Neptune's forks and all kinds of other weird symbols on the
chalkboard that I later learned were related to calculus and other
advanced mathematics.

Well, math just isn't my thing to start with and, furthermore, I was
really shortchanged in elementary and high school by receiving a very
poor grounding in math of any sort.  The few courses I did manage to
complete were just not of the sort to prepare me for the science
courses I really wanted to study. I could handle the theoretical side
of those courses just fine, but one also needed the practical math
skills and those I just didn't have. Of course, I had to drop organic
chemistry, and also rethink a pre-med major. So -- anyway, just
thought I'd mention this in case you wanted mention to your daughter
to make sure she has/gets the math in high school (or early on in
college) to handle those science courses ....

Best wishes to you both,
Byrd
Subject: Re: Medical Doctor Career, Process of
From: snazz-ga on 20 Jan 2005 10:27 PST
 
byrd-ga,

Hi. Here is a little background of me. I am hearing and speech
impaired and I have had the experiences of a wonderful PA-C and a
homecare nurse that we have gotten to know well over the years because
of the many surgeries I have had. I was always interested in medicine
and wanted to become a doctor. Over the years, I doubted whether being
a doctor is right for me. Time, doubtful of my ability to retain
information, my disability, and not being good at math is all being
considered. I also have considered being a scientist when I was in
high school.I have done very well in all aspect of sciences and I
liked it.

Anyway, I liked your comment on medicine and how you weren't good at
math. I am not great at math and I also was shortchanged in middle and
high school. I graduated two years ago. I stayed home to help my
parents who are getting older and was trying to figure out what I
wanted to do.

So, I was wondering what you chose as your pre-med major. Or what did
you decide to do with your science background with the lack of math.
Thank you for taking the time to answer my qustion.

Sincerely, 

snazz-ga
Subject: Re: Medical Doctor Career, Process of
From: byrd-ga on 26 Jan 2005 17:46 PST
 
Hello snazz-ga,

I'm sorry that I didn't see your question sooner. Many thanks to my
esteemed colleague, Boquinha-ga, who originally answered your
question, for alerting me to your most recent comment. I might not
have seen it otherwise. I'll be more than happy to answer your further
questions.

As I had always liked science, I initially chose Biology as my pre-med
major, and did quite well overall until, as I mentioned, I ran into
trouble with the Organic Chemistry and the math requirements that were
quite over my head. While I stayed a while longer with that major, I
ultimately realized that medicine was not the right field for me to
pursue, nor would I succeed well in any Science major because of the
(at that time and place anyway) math requirements for a B.S. in any
scientific field. Eventually, after following a few dead ends, I chose
to major in English, with a Writing minor. Those are areas I've always
had some talent in, and with urging from my academic counselor, it
seemed the easiest course to follow. I did do quite well at it, making
high grades, the Dean's List, etc., which all seemed to confirm that I
had found the right course for me.

In retrospect, I agree it was probably the best path, as I'm really
more of a generalist, with wide-ranging interests, and would feel
constricted if my efforts were confined to a single field. So the
English/Writing concentration has allowed me to pursue research and
writing, which I do very much enjoy, as I can then indulge my
curiosity about a variety of subjects. Earnings outlets have included
freelance writing and research, video production in the legal arena
(which also included quite a bit of research), and being able to work
as a Google Answers Researcher. Also, since I'm able to find out
information, I've been able to teach myself a lot of other useful
skills, which have resulted in earnings opportunities in business and
retail. In addition, I've been able to put those skills to good use in
a number and variety of community volunteer activities.

As to what I did with my Science, well there were several things
actually, that have enriched my life in a number of ways, though
avocationally rather than professionally. I pursued my interest in
Biology by becoming a self-taught amateur botanist and herbalist, with
particular knowledge of plants and herbs of the upper Midwest (U.S.),
and specialized knowledge of the medicinal and food uses of those and
other common herbs. I'm less well versed in local flora now that I
live in the Southern part of the U.S., but have recently taken up the
study again now that my nest is empty and I have more time on my
hands. I use this knowledge for my own benefit and that of my family,
and share it as I can with anyone who is interested. This is my outlet
for the 'healing urge' and desire to help others that originally was
behind my interest in medicine.

And then, I also followed a longheld dream and became a licensed pilot
- ah! that math again! - earning a Commercial certificate with an
Instrument rating for single-engine airplanes. Let me tell you, that
was the most challenging activity I believe I have ever undertaken in
my entire life. It certainly did not come easily to me, especially
absorbing the technical knowledge required, and the considerable math
skills necessary for performing various calculations. I've never had
to work so hard to learn anything. It's only thanks to my
stubbornness, combined with good luck in finding an incredibly patient
and skillful instructor, that I was able to accomplish that goal! And
while I don't make any money at it, I have used the license in civic
volunteer activities of several kinds, including as a Mission Pilot
for the Civil Air Patrol.

All in all, I haven't had an illustrious career as the world counts
it, but I consider that my greatest accomplishment has been
successfully raising my two sons, of whom I am very proud. I am now
also happy to use whatever knowledge and skills I possess to
contribute to the upbringing of my two grandchildren.

Did I have it all to do over again, looking back from the perspective
of more decades than I can believe, I think I would still be best
served by pursuing a field that has allowed me to indulge my
insatiable curiosity in just about any area. However, I also do
believe I would have been well suited to a career in science and
scientific research, though of a type of science that did not require
the math skills I was so short on. The same curiosity and tenacity
that make me good at general research and writing would translate well
to purely scientific pursuits I think, as long as they were
theoretical more than practical. I would not be a good engineer!

As to your search for your direction in life, I heartily commend you
on doing so early what I have not done until much later in life -
taking an honest self-assessment and then deciding to pursue not only
what you would like, but also what you honestly feel you can succeed
and do well at. For that reason, I feel sure you will ultimately
succeed at whatever you decide to undertake, because you are choosing
your course so carefully, and seeking as much input and advice as you
can, early on. That alone is enough to ensure your satisfaction in
living a productive and useful life.

If there is anything else you would like to ask, please do so.  I'll
mark your question and monitor it.  I wish you all the best in your
search for your path.  I feel certain you will find it.

Warmest regards,
Byrd-ga
Subject: Re: Medical Doctor Career, Process of
From: snazz-ga on 28 Jan 2005 12:22 PST
 
Hello Bryd-ga,

Thanks for getting back to me. 

After I wrote to you, I looked on the computer and I found a type of
doctor that I think I might like: naturopathic doctor. It is a mixture
of some allopathic medicine and alternative medicine. It will let me
"prescribe" treatments of massage, herbs, etc. while at the same time
will let me be like a family doctor. It doesn't seem to have too much
math in it. I also like nature and am a hands on person. It takes 4-5
years and it would satisfy my "healing urge." I would have to be
licensed and trained for emergency medicine. Although there are
controversy about alternative medicine, I think I would still go with
it.

I will keep an open mind on choosing English/Writing as my premedical
major if I go this route. I am also "a generalist, with wide-ranging
interests, and would feel constricted if my efforts were confined to a
single field."  I would like to pursue a field that will allow me to
satisfy my curiosity in just about any area.

I am still researching my options. Thank you for your answers. They
have opened up my eyes to the many different paths that can be taken.
Thank you for your encouraging words.

Sincerely, 

snazz-ga
Subject: Re: Medical Doctor Career, Process of
From: snazz-ga on 28 Jan 2005 12:25 PST
 
Oops, I am very sorry that I spelled your name wrong.
Subject: Re: Medical Doctor Career, Process of
From: byrd-ga on 11 Mar 2005 07:07 PST
 
Hello snazz-ga,

I was just perusing your question again, and realized I had not seen
your latest comment.  I am very happy if you found my remarks helpful.
 Your idea of becoming a naturopathic doctor sounds like an excellent
choice to me, for the reasons you stated, but I am happy to see you
are so open-minded about looking into fields you may not have thought
of previously. Whatever course you ultimately choose to follow
professionally can only be enhance by further knowledge of your
options.  Again, with your positive attitude, careful thought, and
high level of self-knowledge and insight, I feel certain you will
succeed in not only finding, but pursuing the best path for you!
Thanks for letting us be a small part of your journey.

Warmest regards,
Byrd-ga

Important Disclaimer: Answers and comments provided on Google Answers are general information, and are not intended to substitute for informed professional medical, psychiatric, psychological, tax, legal, investment, accounting, or other professional advice. Google does not endorse, and expressly disclaims liability for any product, manufacturer, distributor, service or service provider mentioned or any opinion expressed in answers or comments. Please read carefully the Google Answers Terms of Service.

If you feel that you have found inappropriate content, please let us know by emailing us at answers-support@google.com with the question ID listed above. Thank you.
Search Google Answers for
Google Answers  


Google Home - Answers FAQ - Terms of Service - Privacy Policy